2D Green's Function - Bessel function equivalence

Click For Summary
The discussion focuses on demonstrating the equivalence of a specific integral expression involving a 2D Green's function and Bessel functions. The integral is transformed into polar coordinates centered at (x', y'), leading to two separate integrals for different ranges of theta. The user seeks clarity on the necessary change of variables to establish this equivalence, referencing Gradshteyn and Ryzhik for support. They express a desire for a more intuitive understanding of the Bessel K function to aid in making the logical connection. The thread highlights the complexities of relating Green's functions to Bessel functions in the context of wave equations.
RUber
Homework Helper
Messages
1,687
Reaction score
344

Homework Statement


This is not a homework problem per se, but I have been working on it for a few days, and cannot make the logical connection, so here it is:
-- The problem is to show that
##\frac{1}{4\pi} \int_{-\infty}^{\infty} \frac{ e^{-\sqrt{\xi ^2 + \alpha^2 } |y-y'| + i \xi (x-x') }}{\sqrt{\xi ^2 + \alpha^2 }} d\xi= \frac i4 H_0^{(1)}(i \alpha R) = \frac 1{2\pi} K_0(\alpha R) ##
Where ##R=\sqrt{(x-x')^2 + (y-y')^2 } ##.
##\alpha## is the constant wavenumber of a time-harmonic wave.
This is part of the nitty-gritty explanation of the 2D free space Green's function for waves.
Because the answer is a Bessel function, I expect there to be a change to cylindrical coordinates, which I attempt in section 3.

Homework Equations


I have a claim that the equation above is equivalent to
##i \int_{C(\phi)} e ^{ i k r \cos \beta } d\beta ##
with ##C(\phi)## defined by:
## C(\phi) = \left\{ \begin{array}{l l} \displaystyle
x=-|\phi| & y \text{ from } i \infty \text{ to } 0 \\
y=0 & x \text{ from } -|\phi| \text{ to } \pi - |\phi|\\
x= \pi - |\phi| & y \text{ from } 0 \text{ to } -i \infty
\end{array} \right. ##

Referring to Gradshteyn and Ryzhik, \cite{Gradshteyn2000}, this functional form is equivalent to $\rmi \pi H_0^1(kr)$.

The Attempt at a Solution


Attempting to change to polar coordinates centered at (x',y') using ## x = R\cos\theta, y = R\sin\theta## gives:
##\frac{1}{4\pi} \int_{-\infty}^{\infty} \frac{ e^{-\sqrt{\xi ^2 + \alpha^2 } R\sin\theta + i \xi R\cos\theta }}{\sqrt{\xi ^2 + \alpha^2 }} d\xi ## for ##\theta \in [0, \pi]##
and
##\frac{1}{4\pi} \int_{-\infty}^{\infty} \frac{ e^{\sqrt{\xi ^2 + \alpha^2 } R\sin\theta + i \xi R\cos\theta }}{\sqrt{\xi ^2 + \alpha^2 }} d\xi ## for ##\theta \in [ -\pi,0]##.
The reference I have simply says that with an appropriate subsitition, the equivalence can be seen.
I cannot see what change of variables to make from here. I wonder if perhaps there is a more intuitive form of the Bessel K function that might help me see the connection.
Thank you to anyone who might be able to point me in the right direction.
 
Question: A clock's minute hand has length 4 and its hour hand has length 3. What is the distance between the tips at the moment when it is increasing most rapidly?(Putnam Exam Question) Answer: Making assumption that both the hands moves at constant angular velocities, the answer is ## \sqrt{7} .## But don't you think this assumption is somewhat doubtful and wrong?

Similar threads

Replies
3
Views
2K
  • · Replies 2 ·
Replies
2
Views
2K
  • · Replies 14 ·
Replies
14
Views
2K
  • · Replies 19 ·
Replies
19
Views
2K
  • · Replies 1 ·
Replies
1
Views
2K
  • · Replies 4 ·
Replies
4
Views
1K
  • · Replies 8 ·
Replies
8
Views
2K
  • · Replies 8 ·
Replies
8
Views
2K
  • · Replies 40 ·
2
Replies
40
Views
4K
  • · Replies 1 ·
Replies
1
Views
1K